Talk:2015 AMC 10A Problems/Problem 25

The posted solution relies on calculus, and is not appropriate for the AMC contest. Please update to reflect a geometric probability solution (or borrow the one from nsun48 in the discussion of this problem at http://www.artofproblemsolving.com/Forum/viewtopic.php?f=133&t=623881)